Added Sunny's solutions
This commit is contained in:
@ -5,6 +5,7 @@ Consider a mass $m_1$ on top of a pin. \par
|
||||
Due to gravity, the mass exerts a force on the pin at the point of contact. \par
|
||||
For simplicity, we'll say that the magnitude of this force is equal the mass of the object---
|
||||
that is, $m_1$.
|
||||
|
||||
\begin{center}
|
||||
\begin{tikzpicture}[scale=2]
|
||||
\fill[color = black] (0, 0.1) circle[radius=0.1];
|
||||
@ -12,9 +13,10 @@ that is, $m_1$.
|
||||
|
||||
\draw[line width = 0.25mm, pattern=north west lines] (0, 0) -- (-0.15, -0.3) -- (0.15, -0.3) -- cycle;
|
||||
|
||||
|
||||
\draw[color = black, opacity = 0.5] (1, 0.1) circle[radius=0.1];
|
||||
\draw[line width = 0.25mm, pattern=north west lines, opacity = 0.5] (1, 0) -- (0.85, -0.3) -- (1.15, -0.3) -- cycle;
|
||||
|
||||
\draw[line width = 0.25mm, pattern=north west lines, opacity = 0.5]
|
||||
(1, 0) -- (0.85, -0.3) -- (1.15, -0.3) -- cycle;
|
||||
|
||||
\draw[->, line width = 0.5mm] (1, 0) -- (1, -0.5) node[below] {$m_1$};
|
||||
%\draw[->, line width = 0.5mm, dashed] (1, 0) -- (1, 0.5) node[above] {$-m_1$};
|
||||
@ -44,8 +46,8 @@ As you might expect, it is not stable: the rod pivots and falls down.
|
||||
\draw[color = black, opacity = 0.5] (1.2, 0.0) circle[radius=0.1];
|
||||
\draw[-, line width = 0.5mm, opacity = 0.5] (0.7, 0) -- (1.9, 0);
|
||||
|
||||
\draw[line width = 0.25mm, pattern=north west lines, opacity = 0.5] (1.5, 0) -- (1.35, -0.3) -- (1.65, -0.3) -- cycle;
|
||||
|
||||
\draw[line width = 0.25mm, pattern=north west lines, opacity = 0.5]
|
||||
(1.5, 0) -- (1.35, -0.3) -- (1.65, -0.3) -- cycle;
|
||||
|
||||
\draw[->, line width = 0.5mm] (1.2, 0) -- (1.2, -0.5) node[below] {$m_1$};
|
||||
%\draw[->, line width = 0.5mm, dashed] (1.5, 0) -- (1.5, 0.5) node[above] {$f_p$};
|
||||
@ -57,6 +59,7 @@ It therefore exerts a \textit{torque} on the mass-rod system, causing it to rota
|
||||
\pagebreak
|
||||
|
||||
|
||||
|
||||
\definition{Torque}
|
||||
Consider a rod on a single pivot point.
|
||||
If a force with magnitude $m_1$ is applied at an offset $d$ from the pivot point,
|
||||
@ -110,7 +113,6 @@ Find the position of this system's center of mass. \par
|
||||
|
||||
\draw[-, line width = 0.5mm] (-0.5, 0) -- (1.5, 0);
|
||||
|
||||
|
||||
\fill[color = black] (-0.5, 0) circle[radius=0.1];
|
||||
\node[above] at (-0.5, 0.2) {$3$};
|
||||
|
||||
@ -139,9 +141,24 @@ Do the same for the following system, where $m_1$ and $m_2$ are arbitrary masses
|
||||
\end{tikzpicture}
|
||||
\end{center}
|
||||
|
||||
\begin{solution}
|
||||
The CoM will be such that the rod is split into $d_1+d_2=1$
|
||||
according to the relation $m_1 d_1 = m_2 d_2$.
|
||||
This is sufficient, but if you want to solve for one of the $d$,
|
||||
you get $d_1 = \frac{m_2}{m_1+m_2}$.
|
||||
|
||||
\vspace{2mm}
|
||||
|
||||
This should be intuitive; the distance of each mass from the CoM
|
||||
is proportional to the other mass's share of the total mass.
|
||||
\end{solution}
|
||||
|
||||
\vfill
|
||||
\pagebreak
|
||||
|
||||
|
||||
|
||||
|
||||
\definition{}
|
||||
Consider a massless, horizontal rod of infinite length. \par
|
||||
Affix a finite number of point masses to this rod. \par
|
||||
@ -172,5 +189,12 @@ Consider a one-dimensional system of masses consisting of $n$ masses $m_1, m_2,
|
||||
with each $m_i$ positioned at $x_i$. Show that the resulting system always has a unique center of mass. \par
|
||||
\hint{Prove this by construction: find the point!}
|
||||
|
||||
\begin{solution}
|
||||
\begin{equation*}
|
||||
x_0 = \frac{1}{M}\sum_{i=1}^n m_i x_i
|
||||
\end{equation*}
|
||||
where $M = \sum_{i=1}^n m_i$
|
||||
\end{solution}
|
||||
|
||||
\vfill
|
||||
\pagebreak
|
Reference in New Issue
Block a user